Occupational Therapy Certification Exam Practice Test

Reviewed by Editorial Team
The ProProfs editorial team is comprised of experienced subject matter experts. They've collectively created over 10,000 quizzes and lessons, serving over 100 million users. Our team includes in-house content moderators and subject matter experts, as well as a global network of rigorously trained contributors. All adhere to our comprehensive editorial guidelines, ensuring the delivery of high-quality content.
Learn about Our Editorial Process
| By OSUOT
O
OSUOT
Community Contributor
Quizzes Created: 1 | Total Attempts: 39,860
| Attempts: 39,861 | Questions: 200
Please wait...
Question 1 / 200
0 %
0/100
Score 0/100
1. Neurological: A patient just experienced a L ACA infarct CVA, what is the best statement concerning the patient's question about rehabilitation potential?

Explanation

Rationale: It is not appropriate to tell a patient that they will become fully functional or not be able to regain any function, but it is important to remain realistic with the patient about expectations for therapy.
Source: National Occupational Therapy Certification Exam: Review and Study Guide by Rita P. Fleming-Castaldy

Submit
Please wait...
About This Quiz
Occupational Therapy Certification Exam Practice Test - Quiz

Occupational Therapy Certification Exam Practice Test; It is actively responsible for developing, administering, and reviewing the certification process regarding occupational therapy. So, if you are looking towards taking... see morean exam for the same, this quiz has a lot to offer you.

see less

2. General Rehab: Which of the following is a contraindication for electrical stimulation usage?

Explanation

Pacemakers are a contraindication for any electrical stimulation or diathermy usage.
Source- http://www.biomedofla.com/images/contraindications.htm

Submit
3. Pediatrics: A mother reports that her 16 month old has meltdowns whenever water touches her, when she is in public restrooms and hears toilets flushing or hand dryers, when she sees curtains moving at home while windows are open, and whenever she plays with toys that are too visually stimulating. This child most likely has:

Explanation

Answer: B. This child has tactile defensiveness with water, which is a problem associated with overresponsiveness. Overresponsiveness is also known as hyperresponsivity or sensory defensiveness. She has auditory hyperresponsiveness with loud sounds too, and visual overresponsivity with the movement of the curtains. While answer D (gravitational insecurity) is a type of overresponsiveness, this child does not have any signs of gravitational insecurity.

Reference: Case-Smith & O’Brien, Occupational Therapy for Children, 6th edition, pages 346-347

Submit
4. General Rehab: A client with deficits in stereognosis would have MOST DIFFICULTY with?

Explanation

Rational: The correct answer is A because stereognosis is the perceptual skill that enables an individual to identify common objects and geometric shapes through tactile perception without the aide of vision. Stereognosis is essential to occupational performance because the ability to see with the hands is critical to many daily activities.
Source:Text book- Pedretti’s Occupational Therapy: Practice Skills for Physical Dysfunction and the OT Advantage website

Submit
5. Psychology: What is NOT a sign/symptom of schizophrenia?

Explanation

Significant weight loss is a sign of major depression not schizophrenia
reference: psych notes, author: pedersoen, darlene

Submit
6. Pediatrics: A mother reports that her 16 month old has meltdowns whenever water touches her, when she is in public restrooms and hears toilets flushing or hand dryers, when she sees curtains moving at home while windows are open, and whenever she plays with toys that are too visually stimulating. This child most likely has:

Explanation

Answer: B. This child has tactile defensiveness with water, which is a problem associated with overresponsiveness. Overresponsiveness is also known as hyperresponsivity or sensory defensiveness. She has auditory hyperresponsiveness with loud sounds too, and visual overresponsivity with the movement of the curtains. While answer D (gravitational insecurity) is a type of overresponsiveness, this child does not have any signs of gravitational insecurity.

Reference: Case-Smith & O’Brien, Occupational Therapy for Children, 6th edition, pages 346-347

Submit
7. Orthopedic: Immediate precautions following a total hip replacement procedure include which of the following?

Explanation

Answer: a. following a total hip procedure including a total hip replacement the patient is to follow these precautions, no hip flexion beyond 90, no hip rotation, no crossing legs, no adduction and no bending or bringing foot close to hands

Crepeau, E.S., Neistadt, M.E. (1998). Williard and Spackman’s Occupational Therapy Ninth Edition. Baltimore, MD: Lippincott Williams & Wilkins.

Submit
8. Orthopedic: For an individual with RA, treatment should include:

Explanation

All of the choices would benefit a patient with RA. Community resources can include support groups, the Arthritis Foundation, transportation options, etc.

Submit
9. General Rehab: You are seeing a patient in isolation, with a diagnosis of MRSA. What precautions should you take before treating the patient?

Explanation

Rational: Standard precautions include wearing disposable gloves, a face shield and gown when treating a patient in an isolated environment to prevent spreading the disease to other personnel and residents.
Source: http://www.edcp.org/guidelines/mrsa.cfm

Submit
10. Orthopedic: Which of the following is a hip precaution for the first 8 weeks after a hip replacement?

Explanation

Legs ankles should not be crossed, do not bend at the waist, and you should not lie on your side.
Source- http://www.hipsandknees.com/hip/hipprecautions.htm

Submit
11. Pediatrics: Sensory Integration is...

Explanation

Sensory integration begins to occur at conception. It is primarily controlled by the brain. People have a spectrum of different sensory integration levels, it’s not something that either you have or you don’t have.
Source: “Sensory Integration and the Child” by A.Jean Ayers, PhD

Submit
12. Neurological: You are evaluating a 73-year-old patient who presents with rigidity, resting tremor, hypokinesia, and postural instability. What is this patient's most likely diagnosis?

Explanation

Lundy-Ekman, L. (2007). Neuroscience: Fundamentals of rehabilitation. (3rd ed., p. 352). St. Louis: Saunders Elsevier.

Submit
13. General Rehab: You are treating a patient who has experienced decreased oxygenation.  What breathing technique should you instruct the patient in to increase oxygenation?

Explanation

Radomski, M. & Trombly Latham, C. (2008). Occupational Therapy for Physical Dysfunction. (6th ed). Baltimore, MD: Lippincott, Williams & Wilkins, Page 1309.

Submit
14. Orthopedic: Which of the following does NOT describe a proper tip for good body mechanics when sitting?

Explanation

When seated, it is best to be positioned with your knees and hips level to reduce any unnecessary strain on your back.

Submit
15. Orthopedic: The purpose of getting an orthopedic surgical patient up and moving as soon as the doctors approves it is to: (Choose the best answer)

Explanation

Answer: b. it is important to get an orthopedic surgical patient up and moving as soon as the doctors approves it to prevent complications from immobilization and improve healing for the patient

Crepeau, E.S., Neistadt, M.E. (1998). Williard and Spackman’s Occupational Therapy Ninth Edition. Baltimore, MD: Lippincott Williams & Wilkins.

Submit
16. Neurological: Which of the following strategies is NOT an effective psychosocial intervention for clients recovering from a cerebral vascular accident?

Explanation

“C” is the correct answer since focusing on the client’s negative behaviors solely is detrimental to the client’s recovery. The therapist is not allowing the client to make his/her own decisions in their recovery. Incorporating Choices A, B, and D into client sessions provide a more holistic approach.

Source: Cara, E. & MacRae, A. (2005). Psychosocial occupational therapy: A clinical practice (2nd ed.). Clifton Park, NY: Thomson Delmar Learning.

Submit
17. General Rehab: Sensory modulation disorder is one type of SI dysfunction characterized by fluctuating or extremes in responsiveness to the intensity of one or more sensations.  Specific behaviors may include:

Explanation

Resource: Crepeau, E.B., Cohn, E.S., Boyt Schell, B.A., Williard & Spackman's Occupational Therapy, 11th Edition.

Submit
18. Neurological: Which of the following signs and symptoms are the best common factors occurring in Alzheimer's disease?

Explanation

Alzheimer’s disease is characterized by progressive mental deterioration, which primarily includes symptoms such as memory loss, confusion, and disorientation. These symptoms worsen over time and significantly impact daily functioning and quality of life. The other options describe symptoms that can occur in other neurological or psychological conditions but are not the primary indicators of Alzheimer's disease.

Submit
19. General Rehab: During the evaluation of the same COPD patient, the OT sets a short term activity tolerance goal of the pt performing an activity in the 1.0-2.5 METS range. What activity would match that level of exertion?

Explanation

Answer = D (please refer to Radomski & Latham in the OT for Physical Dysfunction, 6th Ed., pg. 1304)

Submit
20. General Rehab: Which of the following methods is the BEST way to evaluate for a hook grasp?

Explanation

The needle would be held with a two point pinch while being threaded. A glass would be held with a cylindrical grasp and the key being place in a lock would be held with a lateral pinch.

Reference: Johnson, Lorch, and DeAngelis: Occupational Therapy Exam Review Guide. ed 3. FA Davis Company, Philadelphia,2006.

Submit
21. Pediatrics: When working with a sensory defensive child, if they are fearful or hesitant when climbing stairs and hold railings tightly, which sensation are they having a reaction to?

Explanation

These children will also dislike tipping their heads backwards, activities that require you to move in space, and swinging on a swing where their feet won’t touch the ground.

Resource: Too loud Too Bright Too fast Too Tight. What to do if you are sensory defensive in an overstimulating world. Sharon Heller, Ph. D

Submit
22. Neurological: You are seeing a patient who shows aggressive behavior and keeps removing his restraints and pulling at his catheter. Pt. lacks short term recall and seems to be acting out based on past events. Pt is unable to complete self feeding and dressing without maximum assistance. What Rancho level is your pt. functioning at?

Explanation

Gutman, S.A., & Schonfeld, A.B.(2009) Screening Adult Neurological Populations, 2nd edition. The American Occupational Therapy Association, Inc. (p 4-7)

Submit
23. Pediatrics: You are an OT working in a pediatric early intervention setting. You have observed a child in the classroom throwing tantrums during activities such as finger painting and playing in sand, the child often is seen playing in the room away from other children and often will not eat during snack time trying to get out of the chair and move away from the table. Based on observation, which sensory modulation problem would you conclude is causing this child's behavior?

Explanation

Case-Smith, O'Brien; Occupational Therapy for Children, 6th Edition, 2010, pg 346.

Submit
24. General Rehab: When transferring a patient an OT can best protect themselves from injury by doing which of the following?

Explanation

Rationale: Keeping the knees bent keeps you in the correct anatomical position for a transfer. All other answers are incorrect, and could possibly result in injury to the individual performing the transfer. You should stand close to the individual, keep the back in a neutral position, and maintain a wide base of support.

Source(s):

Johnson, C. R., Lorch, A., & DeAngelis, T. (2006). Occupational Therapy Examination Review Guide. Philadelphia: F.A. Davis Company

Submit
25. General Rehab: Which of the following evaluation methods can not be used to test manual dexterity/motor function?

Explanation

the Bay Area Functional Performance Eval. is used as phsychological/cognitive battery. All other responses assess manual dexterity/ fine motor coordination and function.

Submit
26. General Rehab: People who are afraid of falling appear to enter a debilitating spiral exhibiting all of the following except:

Explanation

The correct answer is D: People with a fear of falling become less independent.

Zijlstra, R., van Haastregt, J., Rossum, E., van Eijk, J., Yardley, L., Kempen, G. (2007). Interventions to Reduce Fear of Falling in Community-Living Older People: A systematic Review. The American Geriatrics Society. 55:603-615.

Submit
27. General Rehab: Which mobility skills should be accomplished first?

Explanation

bed mobility (hierarchy of mobility skills - bed mobility, mat transfers, bed transfer, toilet & tab transfer)
source: “Occupational Therapy for Physical Dysfunction” M. V. Radomski & C. A. T. Latham p. 819

Submit
28. Neurological:  A patient displays "alien-arm" syndrome, and is unable to attend to the arm to correct it. This patient is most likely presenting with:

Explanation

Laurie Lundy-Ekman (2007). Neuroscience Fundamental for Rehabilitation 3rd Edition. St. Louis, MO: Saunders-Elsvier. (230).

Submit
29. Orthopedic: Your pt presents with hand numbness, mostly at night, pain and paresthesia in the thumb, index, and middle finger.  What nerve compression disorder coincides with these symptoms?

Explanation

Vining-Radomski, M., Trombly-Latham, C. (2008). Occupational therapy for physical dysfunction. Philadelphia, PA: Lippincott Williams & Wilkins. pp 1148.

Submit
30. General Rehab: When working with a patient 1 day post a total shoulder arthroplasty it is appropriate to do:

Explanation

Rationale: It is not appropriate to do resistive exercises or full range of motion exercises one day post surgery due to the risk of dislocation, however to keep the shoulder from freezing Codman’s pendulum exercises are the most appropriate.
Source: National Occupational Therapy Certification Exam: Review and Study Guide by Rita P. Fleming-Castaldy

Submit
31. General Rehab: When completing a home evaluation, what would be the most important suggestion to the patient to prevent falls:

Explanation

Answer: c. Remove throw rugs from floors

Rationale: All are interventions to prevent falls. However, the most important factor when assessing ones home would be to remove throw rugs. This is an immediate intervention that could be implemented at that moment.
Source: National Occupational Therapy Certification Exam Review and Study Guide 5th Edition by Rita P. Fleming-Castaldy (page 319)

Submit
32. Orthopedic: Which of the following would be the LEAST important to stress to a patient following back precautions?

Explanation

when a patient is following back precautions remember no bending, lifting or twisting. (BLT)
Source: Back precautions from Riverside Hospital

Submit
33. General Rehab: Passive stretching to increase ROM should not involve:

Explanation

To increase (and not just maintain)ROM, the limb must be stretched to the point of maximal stretch, which is just a few degrees beyond the point of mild discomfort- this can be assessed by patients verbal or facial indications. The mild discomfort should not linger after release of stretch (if it does, you may have injured the tissue). The most effective stretch is a slow, controlled movement which allows for the tissues to adjust gradually. The stretch should be held for 15-30 seconds. Quick movements are counterproductive and should be avoided, as connective tissue will resist the movement as a protective mechanism.
Reference:
Occupational Therapy for Physical Dysfunction 6th Edition (Mary Vining Radomski & Catherine A Trombly Latham)

Submit
34. General Rehab: When working with a patient with motor control dysfunction an occupational therapist can observe the patient during an assessment of basic activities of daily living and instrumental activities of daily living.  The therapist must observe how problems in motor control affect motor performance.  What other area(s) of patient's performance should the therapist be concerned with?

Explanation

REFERENCE: Occupational Therapy Practice Skills for Physical Dysfunction, Fifth Edition, Pg. 362

Submit
35. General Rehab: If a patient presents to the clinic with a high level of shoulder pain what type of interventions should you avoid initially?

Explanation

Rationale: Strengthening should wait to be done until pain is low so that further pain is not caused.

Submit
36. General Rehab: Per physician referral, an OT in acute care conducts an evaluation with an adult with a new diagnosis of COPD. The OT evaluates the pt and is contemplating possible treatment ideas. The therapist should do the following except:

Explanation

Answer = B (please refer to Radomski & Latham in the OT for Physical Dysfunction, 6th Ed., pg. 1308)

Submit
37. Neurological: You read in a TBI patient's chart that they are at Level III of the Rancho Los Amigos Levels of Cognitive Function Scale.  Which of the following is NOT an appropriate intervention for a patient at this level of cognition?

Explanation

Rationale: A patient at Rancho Level III has a localized response to stimuli and their response is directly related to the type of stimuli they are receiving. At this point you want to stimulate them as much as possible to get consistent and automatic responses (choice A and B). You also want to keep them positioned and provide ROM since they do not have a lot of active movement (choice C). Choice D is beyond their ability at this point, because they are not even able to follow one-step commands at this level. They are simply responding to stimuli.

Source: Occupational Therapy for Physical Dysfunction, Sixth Edition, Radomski and Latham, p. 1048

Submit
38. Neurological: Upon evaluating and working with Mr. Smith following a CVA, you notice he is having difficulty with ADL activities, bumping into objects, misreading literature, and abbreviating scanning while performing various therapeutic activities. Mr. Smith also is reluctant to change head position duing tasks and states "Only one-half of my eyes seem to be working!" You realize Mr. Smith is most likely suffering from:

Explanation

Gutman, S.A., & Schonfeld A.B. (2009). Screening Adult Neurologic Populations, 2nd Edition. The American Occupational Therapy Association; AOTA Press.

Submit
39. General Rehab: Which option below is most important as the INITIAL OT intervention for an individual with a severe form of Guillain-Barré syndrome who has complete paralysis?

Explanation

Rationale: Since the individual has a severe form of Guillain-Barré, the most important initial approach is to protect weak muscles and prevent contractures. Although ADL performance is important for this patient, protecting muscles and maintaining ROM is most important and light ADL training should follow. After strength improves, balance and strengthening interventions can then be used.

Source(s):

Blesedell Crepeau, E. Cohn, E., Boyt Schell, B. A. (2003). Willard and Spackman's Occupational Therapy. Philadelphia: Lippincott Williams & Wilkins.

Submit
40. Pediatrics: A child with tactile defensiveness can probably best tolerate:

Explanation

Rationale: Children with tactile defensiveness do not seem to mind deep touch as it is calming. Light touch is most aversive. They should always see the source of touch and tend to tolerate actively self-applied stimuli better than when it is done passively by a therapist or someone else.
*Source: Occupational Therapy for Children textbook (By Case-Smith) p. 378

Submit
41. Neurological: Which of the following methods is INCORRECT when working with a TBI patient with a Rancho Level IV?

Explanation

D is the correct answer. During this stage of recovery, we want to decrease the patient’s agitation by attempting to normalize the environment (A,B) and. providing consistency and predictability that counters their confusion (C). If a patient becomes agitated or restless during an activity, change the activity or move to another environment. Source: Occupational Therapy for Physical Dysfunction.

Submit
42. Management & Evidence: When arriving to an elderly patient's home who lives with her daughter and son-in-law for a treatment session, the OTL/R notices that the patient's glasses are broken and the patient has new bruises on her forearms around her wrists.  The OTL/R asks the patient once the son-in-law leaves the room what happened and the patient reports that the son-in-law has been physically abusing her for the past two weeks.  What is the most appropriate action the OTL/R should take?

Explanation

Answer: a. Contact supervisor to report suspicious elderly abuse and suggest that Adult Protective Services get involved

Rationale: Role of Occupational therapist is required to suspected or observed cases of elder abuse. Since the patient stated the son-in-law was abusing her and their was proof from the broken glasses and the bruises on her forearms and wrists, the OTL/R is required by law to report this case to Adult Protective Services so they can investigate.
Source: National Occupational Therapy Certification Exam Review and Study Guide 5th Edition by Rita P. Fleming-Castaldy (page 122)

Submit
43. Pediatrics: What would you recommend to parents of an infant who demonstrates low tone, cries a lot, and has a difficult time being soothed?

Explanation

Answer: C
Fleming-Castaldy, Rita. National Occupational Therapy Certification Exam: Review & Study Guide, 5th edition, (2009). Pg. 96-99

Submit
44. General Rehab: A client is hypertonic in their right UE distal to the elbow secondary to a CVA. The OTR facilitates the client into a right side-lying position for therapeutic exercise. What is the primary benefit of facilitating this position?

Explanation

Rationale: The correct answer is B) to reduce tonicity. By side-lying on the affected side, the client is weight bearing through the hypertonic limb which has been shown to be effective in reducing tone. While answers A,C, and D may all be reasons why a client would be in a side-lying position they are not the primary reasons in this scenario.

Pendleton, H.M., Schultz-Krohn, W. (2006). Pedretti’s Occupational Therapy Practice Skills for Physical Dysfunction (6th edition). St. Louis, MO: Mosby Elsevier.

Submit
45. General Rehab: An adult is hospitalized and diagnosed with mild COPD.  In planning for d/c, the pt wants to continue to exercise.  The most appropriate recommendation for the OTR/L to make would be that the pt. participate in:

Explanation

The yoga and stretching program would put the least amount of pressure on the cardio-vascular and pulmonary systems. Also this activity will be monitored by hospital staff rather than personnel who are not familiar with COPD.
Reference: Fleming-Castaldy, R.P. National Occupational Therapy Certification Exam Review and Study Guide; pg 479

Submit
46. Management & Evidence: When working with patients with mental health diagnoses it is important to take the individual's personal desires into accordance when making therapy goals. Which OT assessment is not in accordance with the Model of Human Occupation frame of reference?

Explanation

Allen Cognitive Level Screen (ACLS). The ACLS is an assessment that looks at cognitive disability and predicts how individuals will function in the community, as well as predict unsafe behaviors associated with specific cognitive levels. The SAOF, IC and RC are self-report assessments that allow for personal reflection.

Submit
47. Pediatric:  A child presents with increased tone, which activity would be the most appropriate?

Explanation

Cheatum, B.A. & Hammond, A.A. (2000). Physical activity for improving children’s learning and behavior: A guide to sensory motor development. Chicago: Human Kinetics.

Submit
48. General Rehab: In what age-range are symptoms of multiple sclerosis most likely to emerge?

Explanation

Rational: Individuals may begin to notice symptoms such as an incoordination or paresthesias of limbs, trunk, or face, as well as an overall fatigue.
Source: Willard & Spackman’s Occupational Therapy, v. 10

Submit
49. General Rehab: During the initial ADL evaluation the patient is provided a toothbrush and asked to brush their teeth.  The patient begins to brush their hair with the toothbrush.  Which behavior is this most likely indicative of?

Explanation

ANSWER: b, ideational apraxia is a conceptual deficit seen as an inability to use real objects appropriately.
Occupational Therapy: Practice skills for physical dysfunction 6th edition, chapter 24

Submit
50. Neurological: A person who recently suffered a TBI is aggressive, agitated due to a heightened response and is confused.  What Ranchos level is the patient at?

Explanation

Answer: B – A person who is classified as a Ranchos level four is confused (often times severely) and commonly aggressive and agitated due to their inability to regulate and interpret outside stimulation.

Submit
51. General Rehab: During an evaluation you test the patient's strength and ROM. The patient's right side is weaker than the left within normal limits. The patient's right upper extremity does not flex above shoulder height. You also observe the patient sitting on the edge of bed and reaching for an item on the floor. Based on the observations and using your clinical reasoning skills, what ADL can you suspect that your patient can do?

Explanation

b. LB dressing because in order to dress the lower body, the patient needs to be able to reach to the floor without losing balance.

Submit
52. Neurological: When using NDT Handling Techniques with a client who has had a stroke, Inhibition techniques should be used for which of the following:

Explanation

Radomski & Latham (2008). Occupational Therapy for Physical Dysfunction. Sixth Edition. Chapter 24: Optimizing Motor Behavior Using the Bobath Approach p. 646

Submit
53. Neurological: What is the most common factor for peripheral neuropathy?

Explanation

Laurie Lundy-Ekman (2007). Neuroscience Fundamental for Rehabilitation 3rd Edition. St. Louis, MO: Saunders-Elsvier. (230).

Submit
54. Orthopedic: Which diagnoses is most likely to be associated of the following characteristics? Pain with resisted thumb extension or abduction Positive Finkelstein's test Can be caused by forceful, repetitive motions involving the thumb Pain with activities such as wringing out washrags, opening jars, and using scissors Swelling over the first dorsal compartment of the thumb

Explanation

Rationale: Lateral epicondylitis is a form of tendinitis where there is point tenderness over the lateral epicondyle. There is no thumb involvement. Grip strength is reduced when the elbow is extended and there are complaints of nighttime aching and morning stiffness of the elbow. Carpal Tunnel syndrome is a compression of the median nerve. Symptoms include numbness and tingling in the median nerve distribution of the hand, nighttime numbness, reduced ROM, and reduced grip strength. Cubital Tunnel syndrome deals with ulnar nerve compression in the elbow. Numbness and tingling in the ulnar distribution of the hand is noticed.
Reference:
Fundamentals of Hand Therapy: Clinical Reasoning and Treatment Guidelines for Common Diagnoses of the Upper Extremity (2007)
Edited by Cynthia Cooper

Submit
55. General Rehab: Which of the following would be the LEAST appropriate exercise program for an older adult?

Explanation

D, consider multiple modes of exercise on alternative days to maintain interest and reduce likelihood of muscle injury, joint overuse, pain, fatigue, and boredom.
Source: Pg 120 of study book

Submit
56. Orthopedic: An OTR is evaluating and patient and notices that the patient's PIP joint is hyperextended and the DIP joint is flexed. What best describes this condition?

Explanation

/ Rational: The correct answer is A. Swan neck deformity is when the PIP joint is hyperextended and the DIP joint is in flexion. Boutonniere deformity is when the PIP joint is in flexion and the DIP joint is hyperextended. Mallet deformity is when the DIP joint is flexed/drooping.
Source: Text book from Dr. Flinn’s class- Fundamentals of Hand Therapy by Cynthia Cooper

Submit
57. Pediatrics: An OT designs an evaluation group for adolescents in an eating disorder unit. The activity most useful to assess the clients' task and social skills is:

Explanation

Fleming-Castaldy, Rita. NBCOT Review and Study Guide. Therapy Ed: Evanston, Illinois.

Submit
58. Psychology: An OTR completes an initial evaluation of an inpatient brought to the hospital for exhibiting bizarre behavior.  The patient reports he has powers that allow him to know stories about an object after he has touched it.  The patient is exhibiting:

Explanation

Reference: Cara and MacRae. Psychosocial Occupational Therapy: A clinical practice, 2nd edition. P. 121.

Submit
59. Mangement & Evidence: Working in a SNF setting, you notice a patient is fatigued and seems more weak then usual. After weighing the patient, you determine that they have lost 9 lbs in the past week. What course of action should you take?

Explanation

Rational: Failure to thrive (FTT) is common within the nursing home environment, and it is important that clients demonstrating symptoms of FTT conserve their energy. Nursing personnel and the physician should be notified to modify treatment/medications, and the dietitian should be notified to monitor calorie intake. Client should conserve what energy they have remaining, and physical activities should be discontinued until weight is regained.
Source: www.fpnnoteook.com, presentation/in-service on FTT

Submit
60. Neurological: You are working with Mr. Brown, who recently suffered a CVA and is now a left sided hemiplegic.  While trying to complete ADLs sitting EOB you notice that he is pushing away from his R-side(strong side) and he resists attempts to correct his posture. What should you do to help Mr. Brown maintain a more upright position?

Explanation

Rationale: Mr. Brown has what is known as pusher behavior and because these patients tend to resist hands on attempts to correct their posture, you need to select treatments that manipulate the environment. So reaching for objects toward his R side encourages a weight shift to his stronger side which will bring him more towards midline. It will also be important to use verbal cues.
Source: Occupational Therapy for Physical Dysfunction pg. 1025

Submit
61. Pediatrics: When evaluating a child who is at risk for shunt malfunction, it is MOST important for the therapist to observe for:

Explanation

When evaluating a child who is at risk for shunt malfunction, it is most important for the therapist to observe for irritability and nausea. These symptoms can be indicative of increased intracranial pressure, which is a common sign of shunt malfunction. Increased tone or decreased tone may be associated with other neurological conditions but may not specifically indicate shunt malfunction. Back pain may be present in some cases of shunt malfunction, but it is not as reliable of a symptom as irritability and nausea.

Submit
62. Neurological: You are the treating therapist for recently TBI patient in an acute care hospital. Upon initial evaluation, your patient was given a Glasgow score of 9. Currently your patient is a rancho level 6. Which would be the most effective method of intervention for your clients motivation, cognition, and memory impairments?

Explanation

Neurological Rehabilitation, 5th edition. Umphred, D. A.. Mosey, 2007. Pg 537- 551.

Submit
63. General Rehab: If a patient presents to the clinic in the freezing phase of adhesive capsulitis what type of interventions should be avoided?

Explanation

Rationale: The primary treatment goal of the freezing phase is to prevent motion loss. Overstretching the capsule during the freezing phase may enhance the inflammatory process. For more information see chapter 10 in Cynthia Cooper text “Fundamentals of Hand Therapy.”

Submit
64. Pediatrics: The occupational therapist observes that an 18 month old child is not able to creep more than a few steps because as he looks up, his hips and knees flex and he ends up W sitting with his arms extended and propped forward.  The occupational therapist reports that he demonstrates:

Explanation

Reference: Fleming-Castaldy, Rita. National Occupational Therapy Certification Exam: Review & Study Guide, 5th Edition, (2009), pg-636.

Submit
65. Neurological: Which area of cognition can be described as an impaired association with neurological dysfunction including the lack of knowledge about own physical/cognitive-perceptual impairments and/or their functional implications as well as the inability to anticipate difficulties, recognize error, or monitor performance within the context of an activity?

Explanation

Willard and Spackman page 749. a attention involves several components, for instance the ability to detect/react, the ability to sustain attention for a period of time, the ability to exhibit selective attention or shift attention, and keep track of stimuli during ongoing activity. Willard and Spackman page 755. c motor planning is the ability to execute learned and purposeful activity. Willard and Spackman page 761. And d, problem solving involves planning and concept. Screening adult neurologic populations page 22.

Submit
66. Neurological: A patient with a CVA that has affected the left hemisphere may exhibit all of the following signs except? 

Explanation

b, c, and d are all signs that a person has suffered a CVA that is located in the left hemisphere. OT for physical dysfunction page 1009. a, left hemiparesis is a sign that would be present if someone had suffered a CVA of the right hemisphere. OT for physical dysfunction page 1009.

Submit
67. General Rehab: A patient with macular degeneration is having difficulty with meal preparation. The following are environmental factors to consider to improve the patient's ability to perform meal preparation except:

Explanation

Reference: Occupational Therapy for Physical Dysfunction, Sixth Edition, p. 731-732

Submit
68. Neurological: While completing an evaluation on a stroke patient, you start to notice they are having some visual difficulties. What is the most common visual disturbance that you should be looking for? 

Explanation

Rationale: It is the most common visual disturbance associated with stroke.
Source: Occupational Therapy for Physical Dysfunction pg. 1018

Submit
69. General Rehab: Which one of the following is not a benefit of using Nintendo Wii in burn rehabilitation as an adjunct traditional therapy?

Explanation

Using Nintendo Wii in burn rehabilitation as an adjunct to traditional therapy provides several benefits such as recovering from motor deficits, distraction from pain, and increasing motivation. However, it does not directly address fatigue and soreness. While the use of Nintendo Wii can help improve overall physical function and engagement in therapy, it may not specifically target fatigue and soreness experienced by burn patients.

Submit
70. Psychology: A young female presents to the ER with increased sweating, chest pain, nausea and fear of losing control. She states that this has happened several times in the last 3 months and typically the symptoms subside after 10 minutes. She is scared and wants to make sure she isn't having a heart attack. After doing an EKG and several other tests, cardiac problems are ruled out and more information is sought out about her symptoms. The patient states that as of two months ago she has become scared to be in crowded places and avoids anywhere that has lines or crowds and avoids situations where she knows she will feel this way (supermarkets, movie theaters and buses). What is her diagnosis?

Explanation

Panic Attack and Agoraphobia are NOT codable disorders, the specific disorder in which panic attack or agoraphobia occur are the actual diagnoses so A and D can be ruled out. Because the patient has become increasingly worried about being in places or situations in which she can’t escape when a panic attack occurs and she avoids situations in which she notes panic and anxiety, agoraphobia should be included in the diagnosis, which excludes B as well.

References:
DSM-IV-TR (Panic Disorder Question)
http://www.uptodate.com/patients/content/topic.do?topicKey=~UUVUUsuHJkzvcYr&selectedTitle=1~83&source=search_result (Schizophrenia Question)
http://www.nimh.nih.gov/health/publications/bipolar-disorder/how-does-bipolar-disorder-affect-someone-over-time.shtml (Bipolar Disorder Question)

Submit
71. Orthopedic: An OTR is developing an exercise program for a client with Reflex sympathetic dystrophy/Complex regional pain syndrome in her right hand.  The client is demonstrating pain, swelling, and stiffness.  Which initial exercise plan would be BEST?

Explanation

Rational: The correct answer is B because active range of motion is crucial. Intervention that increases pain such as passive range of motion should be avoided. Massage reduces the swelling and reintroduces touching of the hand.
Source: Text book- Pedretti’s Occupational Therapy: Practice Skills for Physical Dysfunction and the OT Advantage website

Submit
72. General Rehab: An OT designs an evaluation group for adolescents in an eating disorder unit. The activity most useful to assess the clients' task and social skills is:

Explanation

Fleming-Castaldy, Rita. NBCOT Review and Study Guide. Therapy Ed: Evanston, Illinois.

Submit
73. General Rehab: An OTR is developing an exercise program for a client with Reflex sympathetic dystrophy/Complex regional pain syndrome in her right hand.  The client is demonstrating pain, swelling, and stiffness.  Which initial exercise plan would be BEST?

Explanation

Rationale: Active range of motion is crucial. Intervention that increases pain such as passive range of motion should be avoided. Massage reduces the swelling and reintroduces touching of the hand.
Source: Text book- Pedretti’s Occupational Therapy: Practice Skills for Physical Dysfunction and the OT Advantage website

Submit
74. Orthopedic: Which of the following exercises would be a contraindication for a patient with rheumatoid arthritis?

Explanation

Answer: B
High-impact exercises like running should be avoided due to the increased stress it puts on joints. The best exercises for patients with RA are low-impact activities.

Submit
75. Orthopedic: A child is admitted to the hospital with a distal radial buckle fracture.  The doctor asks your opinion on casting vs. splinting of this patient since you are the splinting expert at the facility.  You reply:

Explanation

Source: Plint, A. C., Perry, J. J., Correll, R., Gaboury, I., Lawton, L. (2006). A randomized, controlled trial of removable splinting versus casting for writ buckle fractures in children. Journal of the American Academy of Pediatrics, 117(3), 691-697.

Submit
76. General Rehab: You are seeing a new patient at your SNF who has received a total hip replacement after experiencing a fall at home. The patient is complaining of pain in his right shoulder from when he attempted to catch himself from falling. While reviewing his medical records you see that his RUE radiographs are negative for fracture. You also note that this patient had a pacemaker placed four years ago. Which physical agent modality is not appropriate to use to address this patient's pain?

Explanation

Answer: C. TENS – Electrical modalities are contraindicated with patients who have pacemakers or cardiac conditions.
McHugh-Pendleton, H., & Schultz-Krohn, W. (2006) Pedretti’s Occupational Therapy Practice Skills for Physical Dysfunction Sixth Edition. St. Louis MO: Mosby Inc.

Submit
77. Orthopedic: All of the following are good suggestions to provide a patient who is returning home after hip surgery, except:

Explanation

Kitchen items most often used should be placed at a reachable level, between shoulder and waist level. Following hip surgery, patients should not be bending down for things in low cabinets or shelves. Suggesting the use of a high stool when working at the kitchen counter (A) ensures that the patient is following hip precautions and is a good tactic for conserving energy. Removing throw rugs in the home (B) can help a patient to avoid tripping. Using a rolling basket or utility cart (C) to carry items from place to place will prevent the need for multiple trips when working around the house.

Submit
78. Management & Evidence: When treating patients in an inpatient rehabilitation setting which of the following is the most common assessment used to address daily functioning?

Explanation

the FIM is a basic measure of the need of assistance for a patient it is also used in the Inpatient Rehabilitation Facility-Patient Assessment Instrument (IRF-PAI) which is used to determine length of stay for insurance companies.
Source: Occupational Therapy for Physical Dysfunction p. 82, 83

Submit
79. Psychology: When working in a long term care facility, you need to evaluate the long term memory of a resident; which of the following methods best evaluates memory of personally experienced events?

Explanation

Johnson, Lorch & DenAngelis. Ocuupational Therapy Examination Review Guide. P. 17.

Submit
80. Neurological: A patient presents to your clinic with complaints of persistent numbness in the index and long fingers. She reports that the numbness increases when she drives or does needlepoint. She also reports often waking up at night with pain in her hand. Which is the most likely diagnosis?

Explanation

Reference: Weiss, S. (2005). Hand rehabilitation: A quick reference guide and review. St. Louis, MO: Elsevier Mosby.

Submit
81. Neurological: At what level of spinal cord injury is the important skill of tenodesis grasp maintained?

Explanation

Radomski & Latham (OT for Physical Dysfunction)

Submit
82. Management & Evidence: A client demonstrates impaired fine motor function after sustaining a CVA.  Which assessment would not be appropriate for the OTR to administer to evaluate the client's coordination?

Explanation

Rationale: The correct answer is D) Visual Analog Scale. All of the other choices are assessments that could give you information about a client’s fine motor function. The Visual Analog Scale is designed to assess pain and therefore would not provide any valuable information regarding fine motor function.

Pendleton, H.M., Schultz-Krohn, W. (2006). Pedretti’s Occupational Therapy Practice Skills for Physical Dysfunction (6th edition). St. Louis, MO: Mosby Elsevier.

Submit
83. Neurological: Mrs. Miller is a 31 y/o female diagnosed with a traumatic brain injury.  As the occupational therapist you are using a transfer of training approach to help Mrs. Miller recover from her traumatic brain injury. You want Mrs. Miller to develop and carry out a daily schedule of activities. Which is the MOST appropriate to do during your treatment session?

Explanation

Pendleton, H.M. & Schultz-Krohn, W. (2006). Pedretti’s occupational therapy practices skills for physical dysfunction. St. Louis, MO; Mosby Elsevier.
Fleming-Castaldy, R.P. (2009). National Occupational Therapy Certification Exam Review & Study Guide.

Submit
84. Neurological: A client has been hospitalized for surgical repair of a right-side hip injury.  Her medical history indicates that she had a remote stroke affecting her left side.  Per her chart, this client has been advised that she should not drive due to slow reaction times and homonymous hemianopsia.  Upon discharge from the hospital, she returns to her one-story apartment where she will receive home health occupational therapy.  Which of the following interventions would you NOT incorporate into therapy sessions for this client?

Explanation

“A” should be incorporated into sessions to everyone caring for the client so they can in turn provide a possibly medicated client of her hip precautions. Since she has returned to her familiar home environment, she may attempt to return to “normal” too soon, thus forgetting her precautions. “B” is important in home health OT because these items assist clients with navigating their environment independently, safely, and functionally. “C” should be part of a client’s treatment since she still needs to attend follow-up physician’s appointments, even though she no longer drives. “D” is the correct answer because hip precautions do not allow flexion past 90 degrees.

Source: Fleming-Castaldy, R. (2009). National occupational therapy certification exam: Review & study guid (5th ed.). Evanston, IL: TherapyEd.

Submit
85. Psychology: A middle aged woman comes into the hospital with auditory and visual hallucinations, delusions, disorganized speech and grossly disorganized behavior. She has a history of schizophrenia. What is known about these particular presenting behaviors?

Explanation

These symptoms are known as positive symptoms, which are most responsive to pharmacological treatment. Negative symptoms include loss of affective responsiveness, verbal expression, personal motivation, enjoyment, social drive or attention to environment, basically a decrease or absence of normal characteristics. Negative symptoms show the least amount of change with pharmacological intervention.

Submit
86. Neurological: If a patient has a stroke in the left hemisphere, which of the following will be a deficit?

Explanation

Smith-Gavai, H. Occupational Therapy in Acute Care. Page 186.

Submit
87. Psychology: Conduct disorder is the more serious of the two disorders and ODD is sometimes a precursor to of conduct disorder.

Explanation

Crepeau, E.B., Cohn, E.S., Schell, B.A.B, (2003). Willard & Spackman's Occupational Therapy 10th Ed., Lippincott Williams & Wilkins, PA, p 734-735.

Submit
88. Orthopedic: What is NOT a proactive contribution for a patient who is recovering from a total hip replacement?

Explanation

Sitting on soft chairs is actually not recommended due to the fact that a person can sink lower than knee height, they are not firm and straight back , and are harder to transfer in and out of. All of the other answers are proactive suggestions to help a patient recover.

Submit
89. Pediatrics: The primary role of an Occupational Therapist in sensory integration is...

Explanation

Specific skill attainment won’t aid in integrating one’s skills. You want the child’s senses to be in balance. A sensory integration disorder most certainly doesn’t ostracize a child from their peers.
Source: “Sensory Integration and the Child” by A.Jean Ayers, PhD

Submit
90. General Rehab: Which of the following is not a consideration when working with Oncology patients who have bone metastases?

Explanation

patients are not always on strict bedrest and should be informed of the risks and benefits of both weightbearing and bedrest.

Submit
91. Management & Evidence: According to the evidence, physical environment modifications are most effective for which of the following purposes?

Explanation

The most current evidence suggests that with regards to physical environment modifications, home hazard reductions may be most effective with older adults who have a history of falling. There is growing evidence related to home modifications but much less evidence is available to support environmental modifications in workplace, school, and community environments.
Crepeau, E. B., Cohn, E. S., & Boyt Shell, B. A. (2009). Williard and Spackman’s Occupational Therapy Eleventh Edition. Baltimore, MD: Lippincott Williams & Wilkins.

Submit
92. Psychology: Which is NOT true of both Oppositional Defiant Disorder (ODD) and Conduct Disorder (CD)?

Explanation

Crepeau, E.B., Cohn, E.S., Schell, B.A.B, (2003). Willard & Spackman's Occupational Therapy 10th Ed., Lippincott Williams & Wilkins, PA, p 734-735.

Submit
93. Pediatric: Per parental report or therapist observation, a 5 year old boy fidgets so much that daily routines are interrupted, spins or twirls frequently, and rocks in his chair while performing fine motor tasks.  During evaluation, he wants to try all suspended equipment in the room.  This most likely points to:

Explanation

high threshold vestibular processing, child needs a higher/greater amount of input to register the sensation, therefore seeks out greater input or constant input)
(Source: Sensory Profile and Dr. Case-Smith notes/book)

Submit
94. Psychology:  What are three changes you could make to a living environment to decrease its cognitive demands for patients with dementia?

Explanation

Radomski and Latham, pg. 758

Submit
95. Management & Evidence: A long-term goal for an individual with progressive weakness is for the family to carry out his feeding program. Which statement is the MOST appropriate short-term goal?

Explanation

1. Goals should be functional, measurable, and objective. In addition, short-term goals must relate to the long-term goal being addressed. Answer D meets those criteria. Answer A does not provide measurable criteria, nor does it directly relate to the long-term goal of family training. Answer B, while measurable, does not relate to the long-term goal. Answer C describes the long-term goal of family independence in the feeding program

Borcherding, S. (2005). Documentation manual for writing SOAP notes in occupational therapy. Thorofare, NJ: Slack Inc

Submit
96. Neurological: When positioning a stroke patient in supine with hemiplegia, what is the correct position?

Explanation

Rationale: You place a pillow under the involved scapula and hip to prevent retraction in these areas. You can place an additional pillow alongside the client’s involved leg to keep the hip from externally rotating if necessary. Do not place a pillow directly behind the knee as this may contribute to DVT or knee flexion contracture.
Source: Willard and Spackman 10th ed. Pg 605

Submit
97. Pediatric: Marissa is a typically developing child who is able to demonstrate true sucking while feeding. How old is Marissa?

Explanation

Marissa is able to demonstrate true sucking while feeding, which typically develops around 4 months of age. This indicates that Marissa has reached the developmental milestone of being able to suck effectively during feeding.

Submit
98. Orthopedic: A 50 y/o patient arrives to your clinic with persistent numbness and tingling in the hypothenar eminence and small finger. The signs and symptoms are exacerbated and radiate proximally at night. The patient is likely suffering from?

Explanation

Answer B. Numbness and tingling of the small and ring fingers, hypothenar eminence, and anterior forearm are common signs and symptoms of Ulnar Nerve Compression at the Cubital Tunnel. Patients who sleep with the elbows bent and hand close to body are likely to experience worsening signs and symptoms at night because of the increase pressure to the Cubital Tunnel from sustained elbow flexion.
Cooper, C. Fundamentals of Hand Therapy Clinical Reasoning and Treatment Guidelines for Common Diagnoses of the Upper Extremity. Page 232 - 236.

Submit
99. General Rehab: A 62-year-old widow, who lives alone, is preparing to be discharged from an SNF after four weeks of therapy following a hip replacement. The most important equipment the OT should recommend would be:

Explanation

Source: long handled reachers help patients reach for items without breaking their hip precautions. A wheeled walker would be a PT thing! A bedside commode would not be appropriate for someone who is being discharged home and a weighted hairbrush has nothing to do with hip surgery.

Submit
100. Neurological: You are completing an evaluation on Mr. Collins who is a 73 y/o male with a medical history of hypertension, diabetes mellitus type II, peripheral vascular disease, hyperlipidemia and hyperglycelmia. During the evaluation, Mr. Collins reports having blurry vision, complaints of difficulty reading his newspaper, trouble driving at night and also states that his wife must sort his medications since he can't correctly identify the pills by sight anymore. You further assess Mr. Collins' vision and determine that it is MOST likely that the patient's visual impairment involves:

Explanation

Gutman, S.A. & Schonfeld, A.B. (2009). Screening Adult Neurologic Populations. 2nd Ed. Bethesda, MD: American Occupational Therapy Association, Inc. (p. 51).

Submit
101. Pediatrics:  What is the correct sequence for the Development of Play?

Explanation

-exploratory play happens from 0-2 years, symbolic play 2-4 years, creative play 4-7 years and games with rules and directions 7-12 years.

NBCOT Review and study guide, pg 95

Submit
102. Pediatrics:  What is the correct sequence for the Development of Play?

Explanation

Answer: C
-exploratory play happens from 0-2 years, symbolic play 2-4 years, creative play 4-7 years and games with rules and directions 7-12 years.

NBCOT Review and study guide, pg 95

Submit
103. Neurological: An OT is providing intervention for a client who had a stroke involving Broca's region in the left hemisphere. This client would have difficulty with:

Explanation

: a. expression (client's with Broca's aphasia have slowed labored speech with some apraxia also noted, but they have good receptive language skills unless it is too rapid or lengthy.
Pedretti L, Early MB: Occupational therapy: practice skills for physical dysfunction, ed 5, St. Louis, 2001, Mosby. (Found in the Q & A for the Occupational Therapy Board Examination by Patricia Bowyer and Dorthy Bethea, 2007)

Submit
104. General Rehab: Which of the following is the least important thing to do before seeing a patient in acute care?

Explanation

Answer: B, It is not very important to see if the patient is awake or not. In most cases you will wake the patient up to participate in therapy anyway.

Submit
105. General Rehab: A unique therapy goal to address in a morning ADL with a patient following a recent below the knee amputations is:

Explanation

Care following a recent amputation includes instructing patients on proper limb hygiene (Radomski & Trombly Latham, 2008). Lower body dressing is typically addressed in most patients’ morning ADL; however limb care is unique to patient’s with recent amputations.

Submit
106. Neurological: When working with a patient who was recently involved in a MVA, you notice that he is unable to wrinkle his forehead and has difficulty closing his eyes completely. Which cranial nerve has likely been damaged?

Explanation

The patient's inability to wrinkle his forehead and difficulty closing his eyes completely suggests damage to the facial nerve. The facial nerve controls the muscles of facial expression, including the forehead and eyelid muscles. Damage to this nerve can result from trauma, such as a motor vehicle accident. The other cranial nerves listed do not innervate the muscles involved in these specific actions, making them less likely to be damaged in this scenario.

Submit
107. Orthopedic: After a Total Hip Replacement (posterolateral approach), a patient is permitted to do which of the following:

Explanation

B is the correct answer. Positions of instability include adduction, internal rotation, and flexion. While external rotation is allowed for the posterolateral approach, it is not permitted for patients who have had an anterolateral approach.
Pendleton, H. M., & Schultz-Krohn, W. (Eds.). (2006). Pedretti’s occupational threapy: Practice skills for phyisical dysfunction. St. Louis, MO: Mosby Elsevier. (pp. 1024).

Submit
108. Pediatrics: You are evaluating an infant in an outpatient clinic for feeding problems.  What is the first step in the evaluation process?

Explanation

Schuberth, L.M., Amirault, L.M., and Case-Smith, J. (2010). Feeding intervention. In J. Case- Smith & J. Clifford O’Brien (Eds.), Occupational therapy for children (pp. 446-473). Maryland Heights, MO: Mosby Elsevier

Submit
109. Neurological: Cubital Tunnel syndrome involves compression of the following:

Explanation

Cubital Tunnel Syndrome is the second most common nerve compression syndrome involving pressure at the elbow. Symptoms include numbness and tingling along along the ulnar aspect of the forearm and hand.
National Occupational Therapy Certification Exam: Review and Study Guide, 5th ed, 2009, p 134

Submit
110. General Rehab: Which is not precaution for patient in several weeks after having implanted pacemaker?

Explanation

source; http://www.webmd.com/heart-disease/pacemaker-placement

Submit
111. Orthopedic: You are working with a client with an elbow disarticulation amputation, what loss of function can you expect to see with this injury?

Explanation

Keenan, D., Glover, J. (2006). Pedretti's Occupational Therapy: Practice Skills for Physical Dysfunction (6th ed.). pg. 1103, Table 43-1 Amputation Levels, Functional Losses, an Suggested Prosthetic Components.

Submit
112. Which treatment approach for neurobehavioral deficits is characterized by focusing on specific deficits from the bottom-up, assuming that improving individual component performance will lead to enhanced overall skill?

Explanation

The restorative and remedial approach to treatment for neurobehavioral deficits involves targeting specific deficits from the "bottom-up." This means that therapy focuses on addressing underlying impairments or components of function, with the assumption that improving these individual components will ultimately lead to enhanced overall skill and function.

Submit
113. Neurological: An OT is providing intervention for a client who had a stroke involving Broca's region in the left hemisphere. This client would have difficulty with:

Explanation

ANSWER: a. expression (client's with Broca's aphasia have slowed labored speech with some apraxia also noted, but they have good receptive language skills unless it is too rapid or lengthy.
Pedretti L, Early MB: Occupational therapy: practice skills for physical dysfunction, ed 5, St. Louis, 2001, Mosby. (Found in the Q & A for the Occupational Therapy Board Examination by Patricia Bowyer and Dorthy Bethea, 2007)

Submit
114. Neurological: With the patient's eyes closed, you place your patient's arm with shoulder abducted, elbow flexed at 90 degrees, and palm facing out. The patient is asked to indicate if the elbow is flexed or extended and states it is extended. What sensory deficit is your patient experiencing?

Explanation

Pendleton, H. M., & Schultz-Krohn, W. (Eds.). (2006). Pedretti’s occupational therapy: Practice skills for physical dysfunction. St. Louis, MO: Mosby Elsevier. (520).

Submit
115. Neurological: A patient with a lesion in the cerebellum may present with all of the following symptoms except?

Explanation

a, b, and d are all signs or symptoms that may present with a lesion in the cerebellum. Screening the adult neurologic population page 112. c, apraxia is often times seen in a person whom has a lesion in the frontal lobe. Screening the adult neurologic population page 148.

Submit
116. Neurological: You are treating a client who has sustained a CVA and presents with impaired right UE motor control.  When grading your treatment, what functional task would be the most challenging for the client to work towards?

Explanation

Weightbearing (A) and supported reach (B & C) activities help establish the motor control, motor pattern, and strength needed to support higher levels of functional UE use such as reaching in space (D)
Pendleton, H.M. & Schultz-Krohn, W. (2006). Pedretti’s Occupational Therapy Practice Skills for Physical Dysfunction, Sixth Edition. St Louis, MO: Mosby Elsevier.

Submit
117. Neurological:  The goal for a patient who has had a CVA is to be able to put on a shirt independently. The MOST effective way to structure dressing training for maximum learning retention and generalization of this skill is:

Explanation

Johnson, C., Lorch, A., DeAngelis, T. (2001). The Occupational Therapy Examination Review Guide, Second Edition, pg 118

Submit
118. Pediatric: A child has sensory processing issues, specifically in body awareness and modulation affecting attention and ability to transition throughout the day.  Which of the following is NOT an appropriate intervention for this child?

Explanation

ANSWER: C. Free play/running with peers in an open gymnasium will be too overstimulating for a child with these issues. It will be difficult for them to keep their body under control or focus on what is going on. The child will probably become too overstimulated or shut down. Swinging, jumping, and joint compressions/massage are good activities to help provide regulation and body awareness.

Submit
119. General Rehab: Caution MUST be taken when considering the use of electrical stimulation with which of the following patients?

Explanation

Electrical stimulation should not be applied directly over or in proximity to active cancer. There are no warnings or contraindications for e-stim related to arthritis (B) or broken bones (D). Answer C is the distracter, as patients with uncontrolled high blood pressure should not receive electrical stimulation, but those on medication to regulate hypertension are safe to receive this treatment.

Submit
120. Orthopedic: After a fracture to the proximal 2/3 of humerus, what is a common secondary diagnosis that can decrease a patient's grip strength?

Explanation

Radial Palsy is the most common secondary diagnosis from Humeral fractures. Radial palsy can cause weakness of the extensor muscles of the hand. With weak extensor muscles patients cannot perform full wrist tenodesis which maximizes grip strength.
Cooper, C. Fundamentals of Hand Therapy Clinical Reasoning and Treatment Guidelines for Common Diagnoses of the Upper Extremity. Page 216 - 219.

Submit
121. Neurological: This diagnosis is characterized by rapidly progressing polyneuropathy with symmetric muscle weakening and distal sensory loss. The etiology is unknown, however may occur after an infectious disorder, surgery, or immunization.

Explanation

Fleming-Castaldy, R. National Occupational Therapy Certification Exam, 5th edition. 165-168.

Submit
122. Orthopedic: What is the function of the flexor digitorum superficialis (FDS)?

Explanation

flexor digitorum Superficialis flexes the PIP joints Source: pg 126 study guide

Submit
123. Management & Evidence: Constraint-Induced Therapy (CIT) is designed to promote increased use of a weak or paralyzed arm and has been credited with speeding up the cortical map reorganization process in non-human primates and in humans. CIT is based on a combination of two theories including the dynamic systems theory and the task-oriented approach to motor control acquisition. What is true about the dynamic systems theory?

Explanation

Pendleton, H. M., & Schultz-Krohn, W. (Eds.). (2006). Pedretti’s occupational therapy: Practice skills for physical dysfunction. St. Louis, MO: Mosby Elsevier. (792-793).

Submit
124. Neurological: You get a referral to evaluate an individual's executive functioning following a mild CVA, what are you most likely to assess?

Explanation

Following a mild CVA (cerebrovascular accident), executive functioning is commonly affected. Executive functioning refers to the cognitive processes involved in goal-directed behavior, including initiation and planning. Therefore, when evaluating an individual's executive functioning after a mild CVA, the most likely assessment would be initiation and planning. Orientation, attention, and memory may also be assessed, but they are not specifically related to executive functioning in this context. Job interests and spatial relations are not directly related to evaluating executive functioning after a mild CVA.

Submit
125. General Rehab: Problems in cognitive functioning can be part of both MS and Fibromyalgia.  Which of the following is not a compensatory strategy you would make to patients with either diagnosis for improving cognitive function?

Explanation

Answer: C because pen and paper cognitive exercises are considered a remediation strategy, not a compensatory strategy

Submit
126. Neurological: OT's often times evaluate muscle spasticity. What does muscle spasticity NOT include:

Explanation

Spasticity includes: Changes in response to stretch, DECREASED ability to produce appropriate force for a specific task, increased latency of activation, inability to rapidly turn off muscles, loss of reciprocal inhibition between spastic muscles and their antagonists, changes in the intrinsic properties of muscle fibers, and inability to generate enough antagonist power to overcome spastic muscles.

Neurological Rehabilitation 5th edition. Umphred, 2007. pg.545

Submit
127. General Rehab: Which of the following would be appropriate for an older adult who was recently admitted to a SNF following a recent exacerbation of CHF?

Explanation

Reference – Pedretti’s Occupational Therapy: Practice Skills for Physical Dysfunction (Sixth Edition), pg 1143, 1146

Submit
128. Management & Evidence: When using the Rancho Los Amigos Scale to assess a patient who has sustained a TBI who presents with a heightened level of response, severly confused, and may be aggressive reflects what what level of cognitive function?

Explanation

C: IV Cognitive: Confused-Agitated
a. Confusion
b. Dense PTA
i. Heightened level of activity with decreased information processing abilities
ii. Responding primarily to own internal confusion
iii. Dependent for self care
iv. This is when they wake up
v. Typically no memory of incident
vi. Minimal potential for immediate comprehension
vii. Severely decreased attention span
viii. Can be very challenging
ix. Cry, yell, combative (the cerebral hemisphere is not communicating properly)

Radomski and Latham. (2008). Occupational therapy for physical dysfunction. Sixth Edition. Baltimore, MD. Lippincott, Williams and Wilkins

Submit
129. Psychology: All of the following are examples of Psychosocial approaches to working with clients diagnosed with Parkinson's Disease except:

Explanation

Willard and Spackman’s Occupational Therapy 11th ed.

Submit
130. Orthopedic: Which of the following actions does not violate precautions during the first six weeks following lumbar spine surgery?

Explanation

Typical precautions following lumbar spine surgery include no twisting, no lifting greater than 10 pounds, and no bending. Answers A, B and C violate these rules. Answer D represents an appropriate way to sit in a chair to maintain good body mechanics following spine surgery.

Submit
131. Pediatric: An 8-year-old girl is referred secondary to poor handwriting and the OT notices general low muscle tone during evaluation.  This is particularly noticeable in the hands due to the decreased prominence and visibility of the knuckles.  What activities would be most appropriate to focus on as you begin therapy with this child with a goal of improved handwriting?

Explanation

without the necessary hand strength the child may not be able to develop the appropriate grip, intrinsic movements, or endurance for handwriting at the first or second grade level. Practicing handwriting without improving the underlying skills may lead to future bad habits and poor skills.
(Source: Dr. Case-Smith class notes/book)

Submit
132. General Rehab: Which is NOT an intervention for a person with cancer?

Explanation

diathermy is contraindication of active cancer

source; “Occupational Therapy for Physical Dysfunction” M. V. Radomski & C. A. T. Latham p. 819 http://www.rehabpub.com/issues/articles/2008-01_05.asp

Submit
133. General Rehab: When educating a patient on proper breathing techniques who has recently been diagnosed with COPD, an occupational therapist should tell the patient to inhale through the nose as if smelling roses and to exhale through the mouth...

Explanation

ANSWER: d. This is called pursed lip breathing. It increases the use of the diaphragm and decreases accessory muscle recruitment. They should inhale deeply through the nose and exhale through their mouths with pursed lips (as if they are whistling). Their exhales should take twice as long as their inhales and some resistence from their lips in the exhale should be felt.
Occupational Therapy: Practice skills for physical dysfunction, 6th ed. pg 1151

Submit
134. Orthopedic: Range of motion to the hand of a tetraplegia patient is performed in a specific way to facilitate tenodesis grasp. Which is correct?

Explanation

Answer: A – To promote and facilitate tenodesis grasp passive opening of the fingers in extension and closing of the fingers when extended.

Submit
135. General Rehab: Why is it important not to turn up the oxygen on a person who is a CO2 retainer, even if their oxygen statistics have dropped?

Explanation

If a patient has a chronically low levels of oxygen and high levels of CO2, breathing high levels of oxygen can cause a person to slow down their breathing because the peripheral chemo receptors are being signaled to do so. Since a person's breathing is how they blow off excess CO2, you can see how this could be detrimental to a person who already has a high CO2 level. Source: http://respiratorytherapycave.blogspot.com/2008/06/hypoxic-drive-theory.html.

Submit
136. Pediatric: You are an OT in an early intervention toddler classroom where most of your clients are 2-3 years old.  Which kind of play do you expect to be predominate between these children?

Explanation

ANSWER: C. Preschoolers from 24-36 mo. Predominately have parallel play skills with emerging associative social play as they begin to play with one another.

Source: Case-Smith, J. (2005). Occupational Therapy for Children, 5th ed. St. Louis, MO: Elsevier, Inc.

Submit
137. Orthopedic: A 38 year old women was referred to therapy for lateral epicondylitis of her right dominant UE with a positive Cozen's test.  During the acute phase of treatment, sessions should focus on all but:

Explanation

Answer: D. Strengthening occurs more during the restorative phase once pain is improved.

Source: Fundamentals of Hand Therapy, Cynthia Cooper, pg. 290.

Submit
138. Pediatric: You are evaluating the fine motor skills of preschoolers at age 3 to see if they qualify in motor for special education services.  You will evaluate the children's fine motor skills and your PT colleague will use the same assessment to evaluate the children's gross motor skills.  Which standardized assessment is most appropriate to use?

Explanation

ANSWER: C. The PEDI is not specific to fine and gross motor and is mostly parent interview. The SFA is for ages K-6. The BOT-2 has fine and gross motor scores but is for ages 4.5-14.5. The Peabody is the best choice because it has a fine and gross motor scale and ranges from ages 1-84 months.

Source: Case-Smith, J. (2005). Occupational Therapy for Children, 5th ed. St. Louis, MO: Elsevier, Inc.

Submit
139. Neurological: You are assessing a traumatic brain injury patient and note that the patient is very agitated, moving around in his bed almost non-stop (requiring restraints at times).  The patient is also very confused and follows very few one-step commands.  According to the Rancho Los Amigos Levels of Cognitive Function Scale, your patient is at:

Explanation

Rationale: C is the correct answer because according to the Rancho Scale Level IV is confused, agitated. Agitation is the key in this answer. Level II and III are less responsive and only responding to certain stimuli. By Level V the agitation has subsided, but the confusion remains.

Source: Occupational Therapy for Physical Dysfunction, Sixth Edition, Radomski and Latham, p. 1048

Submit
140. Pediatrics: An occupational therapist is evaluating a 12 month old child due to concerns with self-feeding. What skills should the therapist be looking for to determine if the child's current performance is developmentally appropriate?

Explanation

Case-Smith, O'Brien; Occupational Therapy for Children, 2010 pg 453

Submit
141. Pediatrics: An OT is working with a child that is suspected of having difficulties with visual perceptual skill. Developmentally, the first visual component that needs to be assessed is:

Explanation

D. Oculomotor control, Visual field and visual acuity are considered to be primary visual skills and serve as the foundation as hierarchy of visual perceptual skills.
Mosby's Q & A Review for the Occupational Therapy Board Examination. Patricia Bowyer EdD OTR/L BCN (Author), Dorothy P. Bethea EdD MPA-OTR-L (Author)

Submit
142. Psychology: You are seeing a new patient with a history of depression, hypertension, anxiety, schizophrenia, and coronary artery disease.  During your initial evaluation you notice that he constantly appears to be licking his lips and sticking his tongue out.  Later, as your writing up his evaluation, you realize the athetoid and rhythmic – like movement of his tongue is:

Explanation

Cara, Elizabeth & MacRae, Anne (2005). Psychosocial Occupational Therapy: A Clinical Practice. 2nd Ed. Clifton Park, NY: Delmar, Cengage Learning (p.714-717).

Submit
143. Neurological: True or false: Retention of primitive reflexes is less likely in children with hypotonia than those with hypertonia.

Explanation

Answer: A. True. Children with hypertonia are more likely to retain primitive reflexes. However, delays in the development of postural reactions are a major concern for children with hypotonicity.

Neurological Rehabilitation 5th edition. Umphred, 2007. pg. 399

Submit
144. Pediatrics: You are working with a 4-year-old student referred to OT to develop handwriting skills. Which of the following performance tasks would be appropriate for this age group?

Explanation

Answer: c. These are appropriate characters for a 4 year old to print at this stage in handwriting development. Choice a. appropriate for 2 year olds. Choice b. appropriate for 3 year olds. Choice d. appropriate for 6+ years. (Occupational Therapy for Children, Case-Smith, page 588)

Submit
145. Orthopedic: A patient presents to your outpatient facility several weeks after a recent 2nd digit compression fracture. Surgery was completed and several pins were placed in the finger to provide stability. The digit is noticeably swollen and lacks both passive and active range of motion. Upon evaluation you find that his grip and pinch strength are much lower than those in his none affected hand and that he is unable to make a full fist. What would be your first priority with this patient?

Explanation

Rationale: According to a “clinical pearl” in Cooper’s book, “Reducing edema is almost always the first priority; do this and the client will gain motion.” A digit that is extremely swollen will not tolerate stretch to be placed upon it the way that a non-swollen digit would. With edema in the way the finger will be stiff and unable to bend. Once the edema is gone the patient will gain the ability to bend and then focusing upon increasing ROM and strength will be appropriate. Also, it is important to use modalities that are appropriate for the client. Using ultrasound on an individual with pins could be detrimental as
the deep heat could increase the temperature of the metal pins and burn internal tissues.
Reference:
Fundamentals of Hand Therapy: Clinical Reasoning and Treatment Guidelines for Common Diagnoses of the Upper Extremity (2007)
Edited by Cynthia Cooper

Submit
146. General Rehab: To facilitate increasing independence in lower body dressing in a patient with a TL-spine precautions and wearing a TLSO, which intervention is contraindicated?

Explanation

Answer B, if a patient has TL-spine precautions, they need to wear their TLSO whenever they are out of bed or the head of bed is raised higher than 30 degrees.

Submit
147. General Rehab: A person who is unable to maintain focus as an object moves closer and further away is having difficulty with what visual skill?

Explanation

C is the correct answer. Accomodation is the ability of the eye to adjust focus at different distances while convergence is the ability of the eye to maintain focus as an object moves closer and further away. Saccades are quick eye movements that change fixation from one point to another and allow a person to redirect his or her line of sight. Smooth pursuits are eye movements that maintain continued fixation on a moving target.
Radomski and Latham. (2008). Occupational therapy for physical dysfunction. Sixth Edition. Baltimore, MD. Lippincott, Williams and Wilkins. (pp. 241)

Submit
148. Pediatrics: An occupational therapist is evaluating a 12 month old child due to concerns with self-feeding. What skills should the therapist be looking for to determine if the child's current performance is developmentally appropriate?

Explanation

Case-Smith, O'Brien; Occupational Therapy for Children, 2010 pg 453

Submit
149. Pediatrics: You are a brand-new, on-the-job pediatric occupational therapist. Your first in-take has arrived and you are set to evaluate your very first child as a real pediatric OT. This little boy has just turned 4. He craves deep pressure, is very active, and has no sense of fear. He cannot ride a bike, tends to be clumsy, and has fine motor deficits (i.e. cannot hold a pencil correctly and cannot stay within the lines when coloring). What assessments would you use that would give you the exact information you needed while being the most concise?

Explanation

Actual In-take from week of 11/15; assessment confirmation made by Darcie Scheffler, OTR/L

Submit
150. Pediatric: A child with CP scores in the 10th percentile for her age on the Beery VMI.  To determine her visual perceptual abilities, your next step would be:

Explanation

(Answer: d, completing the visual subtest will remove from the results any influence of physical disability secondary to CP and will allow you to examine her visual perception.)
(Source: Dr. Chisty Nelson class notes, VMI examiners manual)

Submit
151. Neurologic: What is the most common brain tumor?

Explanation

Astrocytoma is the most common brain tumor. This type of tumor originates from astrocytes, which are star-shaped cells that support and nourish nerve cells in the brain. Astrocytomas can be classified into different grades, ranging from low-grade (less aggressive) to high-grade (more aggressive). However, overall, astrocytoma is the most frequently diagnosed brain tumor.

Submit
152. General Rehab: When working with a cardiac care patient in the hospital it is important to educate the patient on:

Explanation

Rationale: It is important in the early stages of care that the patient be clear on what might be expected for them before returning home. Strengthening would not be important in the acute care setting given sterna precautions.

Submit
153. General Rehab: You suspect your client may have a visual field cut and would like to conduct a gross screen to find out.  Which of the following are examples of screens for visual field deficits that could be used in addition to careful observation during ADL task?

Explanation

ANSWER: A. and B. (but remember, these are only gross screens- standardized versions of these tests are included in the biVABA- brain injury visual assessment battery for adults) Reference: Pedretti: OT, Practice Skills for Physical Dysfunction- Fifth Edition, page 404

Submit
154. Pediatrics: When working with children with down syndrome, an appropriate activity to work on with a child who is between 2 and 4 years of age, is:

Explanation

The previous activities are for older aged children, and could be beyond the childs skill level.
Resource: Fine Motor Skills in Children with Down Syndrome. A Guide for Parents and Professionals. Maryanne Bruni, BSc OT (C)

Submit
155. Pediatrics: An OT is working with a child that is suspected of having difficulties with visual perceptual skill. Developmentally, the first visual component that needs to be assessed is:

Explanation

Answer: D Oculomotor control, Visual field and visual acuity are considered to be primary visual skills and serve as the foundation as hierarchy of visual perceptual skills.
Mosby's Q & A Review for the Occupational Therapy Board Examination. Patricia Bowyer EdD OTR/L BCN (Author), Dorothy P. Bethea EdD MPA-OTR-L (Author)

Submit
156. Neurological: There are several complications that can occur after a stroke including all of the following EXCEPT:

Explanation

Anxiety is not a noted common occurrence after a stroke, however depression, shoulder sublaxation and apraxia are very common.
Source- R&L physical function book

Submit
157. General Rehab: When evaluating a client for a wheelchair you determine they are appropriate for a regular/adult wheelchair.  The dimensions of the chair will be _____ inches wide by ______ inches deep by ______ inches high, and the correct hip angle is ____, knee angle is ______ and ankle angle is ______.

Explanation

18 inches wide, 16 inches deep, and 20 inches high with the hips at 90, the knees at 90 and the ankles at 90.

Submit
158. Pediatrics: How does a typical child with a Brachial Plexus injury present when entering an OT clinic for an evaluation?

Explanation

The child often presents as if their arm is hanging out of the socket in a straight manner, with the wrist turned laterally.
Source: http://www.cincinnatichildrens.org/svc/alpha/b/brachial-plexus/health/brachial-plexus.htm

Submit
159. General Rehab: An individual with MS is independent with bathtub transfers using a grab bar. The MOST important self-care recommendation the OT practitioner can make regarding bathing is to:

Explanation

Correct answer is B. Hot water may contribute to fatigue in individuals with MS, and therefore should be avoided. Moderate water temperature is recommended. Bathing in cool water is unnecessary and may cause chilling and increase spasticity. Bathing rather than showering is recommended for individuals with poor balance or standing tolerance such as those with MS or COPD. Bathing at the sink may be recommended for individuals who experience difficulty bending such as those with hip or knee replacements or back pain. However durable medical equipment is typically available for all these diagnoses and a tub bench and hand held shower could be recommended.

Radomski and Trombly: Occupational Therapy for Physical Dysfunction, 6th edition
Chapter 40: Neurodegenerative diseases.

Submit
160. General Rehab: A patient who had a myocardial infarction 2 days ago is beginning phase 1 of cardiac rehabilitation. Which of the following actions is ESSENTIAL for the OTR to do during the patient's initial session?

Explanation

ANSWER: b
References: The NBCOT Official OTR Study guide, pg 46 ; Radomski & Latham, Occupational Therapy for Physical Dysfunction (6th ed.). Pages 1302-1303.

Submit
161.
Neurological: A 90-year-old woman suffered a right CVA and will be receiving therapy in a SNF. Upon evaluation the OT determines that left shoulder is subluxed.  The most appropriate approach for treating the subluxed shoulder is:

Explanation

placing the arm in a position that minimizes constant pulling on the shoulder muscles (from the weight of the arm) is the most appropriate way to treat the subluxed shoulder.

Submit
162. Neurological: After experiencing a TBI your patient is found lying on the ground demonstrating decorticate rigidity as a result of damage to the cerebral hemispheres causing interruption in the corticospinal tracts that transmit voluntary motor messages to all extremities.  How would the UE and LE present in this situation?

Explanation

Pendleton, H. M., & Schultz-Krohn, W. (Eds.). (2006). Pedretti’s occupational threapy: Practice skills for phyisical dysfunction. St. Louis, MO: Mosby Elsevier. (845-847).

Submit
163. Management & Evidence: According to the evidence, which of the following intervention plans is likely to be most effective in preventing falls in older adults?

Explanation

All interventions are appropriate, but evidence has shown that interventions addressing multiple risk factors present in the older adult at risk for falls are most effective in the prevention of falls.
Coppola, S., Elliott, S. J., & Toto, P. E. (2008). Strategies to Advance Gerontology Excellence. Bethesda, MD: AOTA Press.

Submit
164. Neurological: An individual with a traumatic brain injury is impulsive during self-feeding which is exemplified by placing too much food in his mouth at one time. What method would MOST effectively develop safer eating habits?

Explanation

Correct answer is C
Having the individual learn to set down the utensil until the mouth is cleared is a method of imposing a restriction on the behavior which can then develop into an established routine to pace himself during feeding. Cutting the food into smaller pieces and placing the food into separate containers are examples of environmental adaptations not habit development. An impulsive person who eats too fast also will have difficulty counting slowly enough to have his mouth cleared in time to the count of 10 is reached. C is the best answer for behavioral modification.

Radomski and Trombly: Occupational Therapy for Physical Dysfunction, 6th edition
Chapter 29: Optimizing Cognitive Abilities.

Submit
165. Management & Evidence: Prior to discharging a patient with a mental illness back into the community it is important to assess how independent an individual will be with IADLs or ADLS and how much assistance they will need. Which assessment does not evaluate how independent an individual will be within their home following discharge?

Explanation

Feasibility Evaluation Checklist (FEC). The FEC (in the most general sense) measures how acceptable an individual will be as an employee. The KTA, EFPT and KELS all evaluate ADL performance, evaluating cooking, self-care or telephone use in individuals with mental illnesses.

Submit
166. Neurological: When functioning at Rancho Level VI, you would expect a patient to require which of the following levels of assistance for daily living skills:

Explanation

Reference: Fleming-Castaldy, R.P. (2009). National occupational therapy certification exam: Review and study guide. p.156, Table 5-4.

Submit
167. Neurological: What is the most common brain tumor?

Explanation

Umphred’s Neurological Rehabilitation, pgs. 813, 815

Submit
168. Neurological: Damage to the right hemisphere would result in the following symptoms except:

Explanation

Speech is primarily controlled by the left hemisphere of the brain, especially in right-handed individuals. Damage to the right hemisphere typically affects left motor praxis, emotion, and visual-spatial processing.

Submit
169. General Rehab: Your patient has had a decline in ADL performance at the SNF. You notice her endurance has significantly decreased since the last time she was on caseload. Which of the following would you NOT use to increase endurance for ADL performance?

Explanation

Answer: C, increased resistance works on strengthening
Citation: pg. 236 of National Occupational Therapy Certification Exam- Review and Study Guide (TherapyEd)

Submit
170. Neurological: Damage to the right hemisphere would result in the following symptoms except:

Explanation

Neurological Rehabilitation, 5th edition. Umphred, D. A.. Mosey, 2007.

Submit
171. Neurological: What does a score of 9 mean on the Glasgow Coma Scale?

Explanation

B is the correct answer. Scores of 9-12 are moderate, scores 13 and higher are minor, and scores of 8 or lower is severe. The Glasgow Coma scale is out of 15 total points.
Source: Occupational Therapy for Physical Dysfunction

Submit
172. Pediatrics: A mother reports that she is worried that her 14 month old has delayed oral-motor skills due to his eating habits. He holds and tries to eat a cracker but sucks on it more than bites it, consumes soft foods that dissolve in the mouth, and grabs at his spoon but bangs it or sucks on either end of it and does not attempt to bring spoonfuls of food to his mouth. Her 14 month old is at the level of a(n):

Explanation

Answer: C. A 14 month old should be dipping his spoon in his food and bringing spoonfuls of food to his mouth, with a lot of spilling. A 5-7 month old should be taking cereal or pureed baby food off of a spoon, a 6-8 month old should attempt to hold their own bottle, and a 9-13 month old should finger-feed themselves portions of soft table foods and dry cereal.
Reference: Case-Smith & O’Brien, Occupational Therapy for Children, 6th edition, page 453

Submit
173. Orthopedic: You are completing an evaluation of a patient, Ms. Allthatandthensome, who is complaining of intermittent bilateral hand numbness and tingling that happens on a daily basis.  She explains that she has been feeling very clumsy lately because she is constantly dropping small objects at work.  She also reports experiencing nocturnal hand pain and paresthesia which wakes her up at night, but feels better after shaking or massaging her hands.  During your observation, you notice thenar eminence atrophy of both of her hands.  You suspect that Ms. Allthatandthensome's symptoms are consistent with Carpal Tunnel Syndrome (CTS).  Which of the following would assist you in further screening the patient for CTS?

Explanation

Carpal Tunnel Syndrome is the result of median nerve compression in the carpal tunnel of the hand. For answer A, Phalen’s Test (a.k.a. the Wrist Flexion Test) involves placing the wrist in a position of full flexion for 1 minute and asking the patient to describe any changes in sensation during or after this posture. A positive response involves patient report of numbness and tingling in the median nerve distribution of the hand. For answer B, Froment’s sign is observed when there is flexion of the IP joint of the thumb as the flexor pollicis longus attempts to compensate for the paralyzed or weak adductor pollicis and flexor pollicis brevis and is indicative of an ulnar nerve lesion. For answer C, Tinel’s sign occurs when there is gentle percussion along the course of the median nerve and tingling persists for several seconds along the median nerve distribution of the hand. Since both the Phalen’s Test and the Tinel’s sign indicate median nerve compression, answer D is the correct choice.

Submit
174. Pediatrics: A mother reports that she is worried that her 14 month old has delayed oral-motor skills due to his eating habits. He holds and tries to eat a cracker but sucks on it more than bites it, consumes soft foods that dissolve in the mouth, and grabs at his spoon but bangs it or sucks on either end of it and does not attempt to bring spoonfuls of food to his mouth. Her 14 month old is at the level of a(n):

Explanation

Answer: C. A 14 month old should be dipping his spoon in his food and bringing spoonfuls of food to his mouth, with a lot of spilling. A 5-7 month old should be taking cereal or pureed baby food off of a spoon, a 6-8 month old should attempt to hold their own bottle, and a 9-13 year old should finger-feed themselves portions of soft table foods and dry cereal.
Reference: Case-Smith & O’Brien, Occupational Therapy for Children, 6th edition, page 453

Submit
175. General Rehab: Your patient recently had a pace maker inserted and is now in your inpatient rehab unit for general debilitation. What activity should be avoided?

Explanation

When a pace maker is placed the pt. should avoid ROM above 90 degrees and lifting more than 5 lbs for the first 6 weeks after placement

Submit
176. Pediatrics: A 7 year old child on your caseload is presenting with sensory processing deficits.  Which evaluation would be the most appropriate to use?

Explanation

A (SIPT ages 4-8, TSI ages 3-5, Sensory Functions in Infants 1-18months)
Fleming-Castaldy, R. 2009. National Occupational Certification Exam. 5th edition pages 258-259.

Submit
177. General Rehab: You perform a manual muscle test on Nathan a 69 year old patient who demonstrates full ROM and is able to withstand minimal resistance. The results of his manual muscle test show depict Nathan is?

Explanation

Based on the information provided, Nathan is able to withstand minimal resistance during the manual muscle test, indicating that he has good muscle strength. The "+" sign indicates that his muscle strength is slightly above average. Therefore, the correct answer is 3+ (F+).

Submit
178. General Rehab: When advising a patient with a higher level SCI how to provide pressure relief seated in their wheelchair for the first time, which is NOT the safest recommendation?

Explanation

Answer A, because soon after a SCI, patients have trouble regulating blood pressure, and leaning forward can lead to dizziness, nausea and loss of consciousness.

Submit
179. General Rehab: Which of the following statements is not true regarding serial casting in individuals with traumatic brain injury?

Explanation

The casting of more than one joint at a time often leads to skin breakdown as a result of multiple pressure points. It is thus recommended that casting be applied to one joint at a time

Pedretti, L. W., Pendleton, H. M. H., & Schultz-Krohn, W. (2006). Pedretti's occupational therapy: Practice skills for physical dysfunction. St. Louis, Mo: Mosby/Elsevier.

Submit
180. Orthopedic: When adhering to hip precautions after a THR, which of the following would be an appropriate OT treatment?

Explanation

Answer: f. Instruction in & practice the use of AE for lower body dressing and transfer training with tub bench and getting on and off toilet with raised toilet seat Rationale: Role of Occupational therapist is to adhere to hip precautions after surgical repair of hip fractures. In order for the patient to continue adl independence, instructing the pt with lower body dressing using AE allows the patient to complete the tasks being modified independent. In addition, the patient needs to be able to complete tub transfers safely and use a raised toilet seat while adhering to hip precautions. Strengthening of B UE and/or exercises for affected hip could be an appropriate treatment; however, this is not relevant to hip precautions. Source: National Occupational Therapy Certification Exam Review and Study Guide 5th Edition by Rita P. Fleming-Castaldy (page 140)

Submit
181. Neurological: Which of the following would you LEAST likely observe following a CVA occurring in the middle cerebral artery?

Explanation

William and Spackman"s Occupational Therapy, v. 10

Submit
182. Management & Evidence: A child with a brachial plexus injury at your outpatient facility has begun to develop contractures in the left elbow.  Using your clinical judgment based on current research trends, you decide to treat the patient with the following:

Explanation

Source: Sheffler, L. C., Lattanza, L., Hagar, Y., Bagley, A., James, M. A. (2012). The prevalence, rate of progression, and treatment of elbow flexion contracture in children with brachial plexus birth palsy. Journal of Bone and Joint Surgery, 94(5), 403-409.

Submit
183. Neurological: Your client at a SNF is recovering from a right CVA infarct. She presents with severe left hemiplegia, resulting in poor trunk control and flaccidity of her left UE. After assessing scapular movement during PROM, you find that she has no upward rotation of her scapula or external rotation of the humerus. Based on proper handling techniques of a hemiparetic upper extremity, what should be AVOIDED when handling your client's affected UE?

Explanation

Answer: B. Shoulder ROM beyond 90 degrees of flexion and abduction should be avoided unless there is upward rotation of the scapula and external rotation of the humerus.
Woodson, A.M. (2008). Stroke. In M.V. Radomski & C.A.T. Latham (Eds.), Occupational therapy for physical dysfunction (pp. 1001-1041). Baltimore, MD: Lippincott Williams & Wilkins.

Submit
184. Pediatrics: A 7 year old child on your caseload is presenting with sensory processing deficits.  Which evaluation would be the most appropriate to use?

Explanation

Answer: A (SIPT ages 4-8, TSI ages 3-5, Sensory Functions in Infants 1-18months)
Fleming-Castaldy, R. 2009. National Occupational Certification Exam. 5th edition pages 258-259.

Submit
185. Orthopedic: Which provocative test would not be used when evaluating De Quervain's tenosynovitis? 

Explanation

Since De Quervain’s affects the first dorsal compartment ( Abductor pollicis longus and extensor pollicis brevis are there), swelling and pain in that area (think: snuffbox, radial styloid area) are indicators. Finkelstein’s is the test where they grab their thumb with their fingers, then move hand toward ulnar deviation- pain in the radial styloid/ radial wrist/thumb area is a positive test. Cozen’s test is a test of the elbow, which is not directly involved in De Quervain’s.

Fundamentals of hand therapy (Cynthia Cooper)

Submit
186. Orthopedic: Which of the following would NOT be therapeutic priorities following an above-knee amputation?

Explanation

Rational: A lower extremity amputation alters one’s center of balance, therefore standing balance and endurance should be a priority. Upper extremity strengthening is important due to increase dependence on the arms following a lower limb amputation. Functional mobility will certainly change and should be a priority in occupational therapy as well as in physical therapy. One would not expect to see a change in grooming techniques as grooming is performed using the upper extremities, which are not impaired in this case.
Source: Willard & Spackman’s Occupational Therapy, v. 10

Submit
187. Management & Evidence: OT's commonly assess peripheral nerve function with screening tests. The best way to test for radial nerve function would be to have the patient perform:

Explanation

To test for radial nerve function, the best way would be to have the patient perform wrist extension. The radial nerve is responsible for extending the wrist, so testing this movement can provide information about the function of the nerve. The other options, gross grasp with Dynomometer, pincer grasp with index and thumb, and thumb opposition, are not specific to radial nerve function and may involve other nerves or muscle groups.

Submit
188. Neurological: A patient has been diagnosed with a right hemisphere cerebral vascular accident, of the symptoms listed below which would NOT be a deficit this patient would have?

Explanation

C- Bilateral motor praxis deficits occur with left CVA.

National Occupational Therapy Certification Exam- Review & Study Guide (5th Edition)
Rita P. Fleming-Castaldy
Copyright © 2009 International Educational Resources, Ltd.

Submit
189. General Rehab: Which of the following is not a possible contraindication for ultrasound use?

Explanation

Rationale: Application of heat or cold directly over pregnant uterus is contraindicated but application to other areas is not; Impaired cognition may affect ability to communicate discomfort; application of heat or cold directly over a metastasis is contraindicated because it could potentially increase circulation to the area so might enhance growth or spread malignancy to other tissues; application to scar tissue promotes the healing process and can facilitate resolution of inflammatory phase of healing
*Information obtained from OT 743 “Assessment of Physical Function in OT” course packet.

Submit
190. Neurological: How can a therapist differentiate between visual field cuts and unilateral neglect?

Explanation

Laurie Lundy-Ekman (2007). Neuroscience Fundamental for Rehabilitation 3rd Edition. St. Louis, MO: Saunders-Elsvier. (230).

Submit
191. Neurological: Which of the following are you least likely to observe following a CVA occurring in the middle cerebral artery?

Explanation

Rational: Symptoms of a CVA occurring at the middle cerebral artery include contralateral hemiplegia, contralateral hemianopsia, sensory deficits, and language deficits.
Source: Willard & Spackman’s Occupational Therapy, v. 10

Submit
192. Neurological: Which of the following would you LEAST likely observe following a CVA occurring in the middle cerebral artery?

Explanation

William and Spackman"s Occupational Therapy, v. 10

Submit
193. Orthopedic: The supraspinatus is one of four muscles part of the rotator cuff. What is the action of this muscle?

Explanation

The supraspinatus muscle is responsible for abducting the arm and stabilizing the humeral head in the glenoid cavity. This means that it helps to move the arm away from the body and keeps the head of the upper arm bone in place within the shoulder joint.

Submit
194. General Rehab: Handling techniques used in the NDT approach are utilized to obtain all of the following except

Explanation

Answer: D, NDT techniques are used to provide external stability during movement, normalize movement patterns, facilitate or inhibit specific muscle groups, inhibit abnormal patterns of control, provide sensory input, increase range of motion, dissociate body segments, and normalize tone. NDT tries to decrease associated reactions by assessing the situations that evoke them and reducing the demand of the task.

Fleming-Castaldy, R.P. (2009). National Occupational Therapy Certification Exam: Review and Study Guide. Evanston, IL: Therapy Ed.

Submit
195. Orthopedic: An OT practitioner is fabricating a static splint that will assist with the maintenance of a functional hand and finger position while keeping the soft tissures of the hand in midrange position. Which splint would the OT MOST likely select to address these needs?

Explanation

It is the most appropriate to fabricate to maintain a functional hand position.
The bivalve cast is typically used when circumferential pressure of a body part is required to maintain a desired position.
The dynamic extension splint is not considered to be a static splint and incorporates outriggers to maintain a functional hand position. A wrist cockup splint does not impact the position of the entire hand since it ends at the MCP crease.

Reference:Pedretti, LW and Early, ME: Occupational Therapy:Practice Skills for Physical Dysfunction, ed 5. CV Mosby, St. Louis, 2001.

Submit
196. General Rehab: An individual had a coronary artery bypass graft 5 days ago, his cardiologist approved his activities at a MET level of 2-3.  What would be an appropriate activity for this individual?

Explanation

An individual who had a coronary artery bypass graft 5 days ago and has been approved for activities at a MET level of 2-3 needs to engage in activities that are light to moderate in intensity. Among the options provided, grooming while standing at the sink would be the most appropriate activity. This activity requires minimal exertion and can be performed while maintaining a comfortable standing position, allowing the individual to gradually increase their activity levels without putting excessive strain on their cardiovascular system.

Submit
197. General Rehab: Which of the following is an appropriate assessment to administer to a teenager (13-18 years old)?

Explanation

Rationale: VMI = ages 2 years 0 months – 18 years 11 months
Peabody = 1 month – 71 months (5 years 11 months)
SIPT = 4 years – 8 years 11 months
Battelle = birth – 8 years
*Source: test manuals

Submit
198. General Rehab: Which of the following would be the LEAST important to stress to a patient following open heart surgery?

Explanation

D, patient should not lift excessively over 90 degrees shoulder flexion but shoulder ROM to 90 should be encouraged to keep shoulder from freezing.

Submit
199. Neurological: An adult incurred an injury to the anterior spinal artery at the T12 level. Upon evaluation, the OT determines that the individual has retained the sensation of:

Explanation

Proprioception is maintained with the condition of anterior spinal cord syndrome because the dorsal column transmits proprioceptive information. The other senses would be impaired

From National Occupational Therapy Certification Exam Review and Study guide by: Fleming-Castaldy

Submit
200. Neurological: You are evaluating a patient whose chart says he has been acutely diagnosed with a Brown-Sequard spinal cord injury. What deficits can you expect to see during your evaluation?

Explanation

Pendleton, H. M., & Schultz-Krohn, W. (Eds.). (2006). Pedretti’s occupational therapy: Practice skills for physical dysfunction. St. Louis, MO: Mosby Elsevier. (906).

Submit
View My Results

Quiz Review Timeline (Updated): Jul 4, 2024 +

Our quizzes are rigorously reviewed, monitored and continuously updated by our expert board to maintain accuracy, relevance, and timeliness.

  • Current Version
  • Jul 04, 2024
    Quiz Edited by
    ProProfs Editorial Team
  • Aug 22, 2012
    Quiz Created by
    OSUOT
Cancel
  • All
    All (200)
  • Unanswered
    Unanswered ()
  • Answered
    Answered ()
Neurological: A patient just experienced a L ACA infarct CVA, what is...
General Rehab: Which of the following is a contraindication for...
Pediatrics: A mother reports that her 16 month old has meltdowns...
General Rehab: A client with deficits in stereognosis would have MOST...
Psychology: What is NOT a sign/symptom of schizophrenia?
Pediatrics: A mother reports that her 16 month old has meltdowns...
Orthopedic: Immediate precautions following a total hip replacement...
Orthopedic: For an individual with RA, treatment should include:
General Rehab: You are seeing a patient in isolation, with a diagnosis...
Orthopedic: Which of the following is a hip precaution for the first 8...
Pediatrics: Sensory Integration is...
Neurological: You are evaluating a 73-year-old patient who presents...
General Rehab: You are treating a patient who has experienced...
Orthopedic: Which of the following does NOT describe a proper tip for...
Orthopedic: The purpose of getting an orthopedic surgical patient up...
Neurological: Which of the following strategies is NOT an effective...
General Rehab: Sensory modulation disorder is one type of SI...
Neurological: Which of the following signs and symptoms are the best...
General Rehab: During the evaluation of the same COPD patient, the OT...
General Rehab: Which of the following methods is the BEST way to...
Pediatrics: When working with a sensory defensive child, if they are...
Neurological: You are seeing a patient who shows aggressive behavior...
Pediatrics: You are an OT working in a pediatric early intervention...
General Rehab: When transferring a patient an OT can best protect...
General Rehab: Which of the following evaluation methods can not be...
General Rehab: People who are afraid of falling appear to enter a...
General Rehab: Which mobility skills should be accomplished first?
Neurological:  A patient displays "alien-arm" syndrome, and is...
Orthopedic: Your pt presents with hand numbness, mostly at night, pain...
General Rehab: When working with a patient 1 day post a total shoulder...
General Rehab: When completing a home evaluation, what would be the...
Orthopedic: Which of the following would be the LEAST important to...
General Rehab: Passive stretching to increase ROM should not involve:
General Rehab: When working with a patient with motor control...
General Rehab: If a patient presents to the clinic with a high level...
General Rehab: Per physician referral, an OT in acute care conducts an...
Neurological: You read in a TBI patient's chart that they are at Level...
Neurological: Upon evaluating and working with Mr. Smith following a...
General Rehab: Which option below is most important as the INITIAL OT...
Pediatrics: A child with tactile defensiveness can probably best...
Neurological: Which of the following methods is INCORRECT when working...
Management & Evidence: When arriving to an elderly patient's home...
Pediatrics: What would you recommend to parents of an infant who...
General Rehab: A client is hypertonic in their right UE distal to the...
General Rehab: An adult is hospitalized and diagnosed with mild COPD....
Management & Evidence: When working with patients with mental...
Pediatric:  A child presents with increased tone, which activity...
General Rehab: In what age-range are symptoms of multiple sclerosis...
General Rehab: During the initial ADL evaluation the patient is...
Neurological: A person who recently suffered a TBI is aggressive,...
General Rehab: During an evaluation you test the patient's strength...
Neurological: When using NDT Handling Techniques with a client who has...
Neurological: What is the most common factor for peripheral...
Orthopedic: Which diagnoses is most likely to be associated of the...
General Rehab: Which of the following would be the LEAST appropriate...
Orthopedic: An OTR is evaluating and patient and notices that the...
Pediatrics: An OT designs an evaluation group for adolescents in an...
Psychology: An OTR completes an initial evaluation of an inpatient...
Mangement & Evidence: Working in a SNF setting, you notice a...
Neurological: You are working with Mr. Brown, who recently suffered a...
Pediatrics: When evaluating a child who is at risk for shunt...
Neurological: You are the treating therapist for recently TBI patient...
General Rehab: If a patient presents to the clinic in the freezing...
Pediatrics: The occupational therapist observes that an 18 month old...
Neurological: Which area of cognition can be described as an impaired...
Neurological: A patient with a CVA that has affected the left...
General Rehab: A patient with macular degeneration is having...
Neurological: While completing an evaluation on a stroke patient, you...
General Rehab: Which one of the following is not a benefit of using...
Psychology: A young female presents to the ER with increased sweating,...
Orthopedic: An OTR is developing an exercise program for a client with...
General Rehab: An OT designs an evaluation group for adolescents in an...
General Rehab: An OTR is developing an exercise program for a client...
Orthopedic: Which of the following exercises would be a...
Orthopedic: A child is admitted to the hospital with a distal radial...
General Rehab: You are seeing a new patient at your SNF who has...
Orthopedic: All of the following are good suggestions to provide a...
Management & Evidence: When treating patients in an inpatient...
Psychology: When working in a long term care facility, you need to...
Neurological: A patient presents to your clinic with complaints of...
Neurological: At what level of spinal cord injury is the important...
Management & Evidence: A client demonstrates impaired fine motor...
Neurological: Mrs. Miller is a 31 y/o female diagnosed with a...
Neurological: A client has been hospitalized for surgical repair of a...
Psychology: A middle aged woman comes into the hospital with auditory...
Neurological: If a patient has a stroke in the left hemisphere, which...
Psychology: Conduct disorder is the more serious of the two disorders...
Orthopedic: What is NOT a proactive contribution for a patient who is...
Pediatrics: The primary role of an Occupational Therapist in sensory...
General Rehab: Which of the following is not a consideration when...
Management & Evidence: According to the evidence, physical...
Psychology: Which is NOT true of both Oppositional Defiant Disorder...
Pediatric: Per parental report or therapist observation, a 5 year old...
Psychology:  What are three changes you could make to a living...
Management & Evidence: A long-term goal for an individual with...
Neurological: When positioning a stroke patient in supine with...
Pediatric: Marissa is a typically developing child who is able to...
Orthopedic: A 50 y/o patient arrives to your clinic with persistent...
General Rehab: A 62-year-old widow, who lives alone, is preparing to...
Neurological: You are completing an evaluation on Mr. Collins who is a...
Pediatrics:  What is the correct sequence for the Development of...
Pediatrics:  What is the correct sequence for the Development of...
Neurological: An OT is providing intervention for a client who had a...
General Rehab: Which of the following is the least important thing to...
General Rehab: A unique therapy goal to address in a morning ADL with...
Neurological: When working with a patient who was recently involved in...
Orthopedic: After a Total Hip Replacement (posterolateral approach), a...
Pediatrics: You are evaluating an infant in an outpatient clinic for...
Neurological: Cubital Tunnel syndrome involves compression of the...
General Rehab: Which is not precaution for patient in several weeks...
Orthopedic: You are working with a client with an elbow...
Which treatment approach for neurobehavioral deficits is characterized...
Neurological: An OT is providing intervention for a client who had a...
Neurological: With the patient's eyes closed, you place your patient's...
Neurological: A patient with a lesion in the cerebellum may present...
Neurological: You are treating a client who has sustained a CVA and...
Neurological:  The goal for a patient who has had a CVA is to be...
Pediatric: A child has sensory processing issues, specifically in body...
General Rehab: Caution MUST be taken when considering the use of...
Orthopedic: After a fracture to the proximal 2/3 of humerus, what is a...
Neurological: This diagnosis is characterized by rapidly progressing...
Orthopedic: What is the function of the flexor digitorum superficialis...
Management & Evidence: Constraint-Induced Therapy (CIT) is...
Neurological: You get a referral to evaluate an individual's executive...
General Rehab: Problems in cognitive functioning can be part of both...
Neurological: OT's often times evaluate muscle spasticity. What does...
General Rehab: Which of the following would be appropriate for an...
Management & Evidence: When using the Rancho Los Amigos Scale to...
Psychology: All of the following are examples of Psychosocial...
Orthopedic: Which of the following actions does not violate...
Pediatric: An 8-year-old girl is referred secondary to poor...
General Rehab: Which is NOT an intervention for a person with cancer?
General Rehab: When educating a patient on proper breathing techniques...
Orthopedic: Range of motion to the hand of a tetraplegia patient is...
General Rehab: Why is it important not to turn up the oxygen on a...
Pediatric: You are an OT in an early intervention toddler classroom...
Orthopedic: A 38 year old women was referred to therapy for lateral...
Pediatric: You are evaluating the fine motor skills of preschoolers at...
Neurological: You are assessing a traumatic brain injury patient and...
Pediatrics: An occupational therapist is evaluating a 12 month old...
Pediatrics: An OT is working with a child that is suspected of having...
Psychology: You are seeing a new patient with a history of depression,...
Neurological: True or false: Retention of primitive reflexes is less...
Pediatrics: You are working with a 4-year-old student referred to OT...
Orthopedic: A patient presents to your outpatient facility several...
General Rehab: To facilitate increasing independence in lower body...
General Rehab: A person who is unable to maintain focus as an object...
Pediatrics: An occupational therapist is evaluating a 12 month old...
Pediatrics: You are a brand-new, on-the-job pediatric occupational...
Pediatric: A child with CP scores in the 10th percentile for her age...
Neurologic: What is the most common brain tumor?
General Rehab: When working with a cardiac care patient in the...
General Rehab: You suspect your client may have a visual field cut...
Pediatrics: When working with children with down syndrome, an...
Pediatrics: An OT is working with a child that is suspected of having...
Neurological: There are several complications that can occur after a...
General Rehab: When evaluating a client for a wheelchair you determine...
Pediatrics: How does a typical child with a Brachial Plexus injury...
General Rehab: An individual with MS is independent with bathtub...
General Rehab: A patient who had a myocardial infarction 2 days ago is...
Neurological: A 90-year-old woman suffered a right CVA and will be...
Neurological: After experiencing a TBI your patient is found lying on...
Management & Evidence: According to the evidence, which of the...
Neurological: An individual with a traumatic brain injury is impulsive...
Management & Evidence: Prior to discharging a patient with a...
Neurological: When functioning at Rancho Level VI, you would expect a...
Neurological: What is the most common brain tumor?
Neurological: Damage to the right hemisphere would result in the...
General Rehab: Your patient has had a decline in ADL performance at...
Neurological: Damage to the right hemisphere would result in the...
Neurological: What does a score of 9 mean on the Glasgow Coma Scale?
Pediatrics: A mother reports that she is worried that her 14 month old...
Orthopedic: You are completing an evaluation of a patient, Ms....
Pediatrics: A mother reports that she is worried that her 14 month old...
General Rehab: Your patient recently had a pace maker inserted and is...
Pediatrics: A 7 year old child on your caseload is presenting with...
General Rehab: You perform a manual muscle test on Nathan a 69 year...
General Rehab: When advising a patient with a higher level SCI how to...
General Rehab: Which of the following statements is not true regarding...
Orthopedic: When adhering to hip precautions after a THR, which of the...
Neurological: Which of the following would you LEAST likely observe...
Management & Evidence: A child with a brachial plexus injury at...
Neurological: Your client at a SNF is recovering from a right CVA...
Pediatrics: A 7 year old child on your caseload is presenting with...
Orthopedic: Which provocative test would not be used when evaluating...
Orthopedic: Which of the following would NOT be therapeutic priorities...
Management & Evidence: OT's commonly assess peripheral nerve...
Neurological: A patient has been diagnosed with a right hemisphere...
General Rehab: Which of the following is not a possible...
Neurological: How can a therapist differentiate between visual field...
Neurological: Which of the following are you least likely to observe...
Neurological: Which of the following would you LEAST likely observe...
Orthopedic: The supraspinatus is one of four muscles part of the...
General Rehab: Handling techniques used in the NDT approach are...
Orthopedic: An OT practitioner is fabricating a static splint that...
General Rehab: An individual had a coronary artery bypass graft 5 days...
General Rehab: Which of the following is an appropriate assessment to...
General Rehab: Which of the following would be the LEAST important to...
Neurological: An adult incurred an injury to the anterior spinal...
Neurological: You are evaluating a patient whose chart says he has...
Alert!

Advertisement